ChaseDream
搜索
12
返回列表 发新帖
楼主: sandy07627
打印 上一主题 下一主题

[求助]prep2-35

[复制链接]
11#
发表于 2016-10-1 16:16:09 | 只看该作者
本题结论是 inappropriate prescriptions would not become more common
因为inappropriate prescription这个动作的施加者是医生,所以b选项病人怎么怎么样与此无关,不能病人去医生或不去医生那里,inappropriate prescription就减少了
12#
发表于 2017-7-19 16:54:14 | 只看该作者
这道题简单看感觉是个who has the final say的问题,文中说physicians have the final say,所以patients have the final say可以削弱。
13#
发表于 2017-11-1 16:55:39 | 只看该作者
看到上面有人说B
B首先就能被排除,因为它说的是some people,这种无意义的概率,立刻就能被排除。
看到minority,some的选项基本一眼就能杀掉
14#
发表于 2019-7-24 11:39:02 | 只看该作者
这题我也错了T T不过后来好像找到问题所在了。
重点是注意前一句!!!不要只看最后一句
论断是:病人会要求医生开不合理的药,但是医生的职业能力会组织病人开不合理的药
B无关
D医生的职业能力和从哪获取广告无关,就算广告误导,他们也应该能够判断的出来
E当自己的药方无效时,医生是否会屈从
15#
发表于 2019-7-24 14:33:31 | 只看该作者
sandy07627 发表于 2008-8-5 08:17
35.   (31595-!-item-!-188;#058&006404) It is illegal to advertise prescription medica ...

P1: Beside directly advertise prescription medication in Hedland to physicians either by mail or medical journal, it is illegal.

P2: law would allow general advertising of prescription medication

P3: Being objected that general population lacks the knowledge to evaluate the advertisement and might ask for inappropriate medications,

P4: Physicians have the final say whether to prescribe a medication for a patient

C: Inappropriate prescriptions would not become more common.

What is the possible point at issue ?

1. Argument presumes for those general population " with " specialized knowledge to evaluate advertisement, they still would ask for inappropriate medication, since they do have other purpose intended the matter that does not exist.

2. Argument also presumes that there is no way patient could do anything they could to either entice or threat, or both entice and threat their physicians for getting the inappropriate medications.

So, the above 2 points are necessary assumptions that must be happened if the original argument must be correct.

All we have to just looking for the answers of options that if yes, could logically be legally established, if no, it could work just as how we do at necessary assumption questions ( Negate the logical stand )

If original argument is correct ( Physician has final say ---> No become more common ), then both point 1 and point 2 mentioned must be existed.

So, if negate point 1 or negate point 2, original argument must be refuted.

A. Yes, they could be alert - well, being effective or not does not logically relevant to being inappropriate. No, they could not be alert - Well, then they just do not know if the effective treatment is on the market. It also does not mean they would get the inappropriate treatment.

B. Yes, they will - Well, it does not mean physician would give to them even if they ask, remember, physician does have " final say " . No, they wont - If they wont go to physician for asking for the inappropriate treatment, then there is no way for them to get it.

C. Yes it does - It only means that there might be more marketing channels to have general public receive the infos, but it does not mean that physician would say yes to the patients even if they have different ways of knowing inappropriate treatments. No, it does not. - Well, its not really relevant. is it ?

D. Whether it is an important sources does not mean physicians does " not " know whether the treatment for the patient is inappropriate.

As long as I could receive the information about the new drugs, and I know its not appropriate for the patients, I would not prescribe the drug for him or her.

" It is not important does not mean the physician can't not receive the infos of new drugs, and if physician still can receive the infos of new drugs, they still do have the final say to prescribe the drugs to the people.


E. Correct answer. It perfectly match our point 2.

Regardless of the fact that whether the original is effective for not, as long as the physicians would give in to the demands from the patients, then it would be inappropriate, which is, there is no final say for them.

If they wont give in, then the original argument would not be refuted.


* Work it as the way you work in the necessary assumption questions, then it would be super easy.
您需要登录后才可以回帖 登录 | 立即注册

Mark一下! 看一下! 顶楼主! 感谢分享! 快速回复:

手机版|ChaseDream|GMT+8, 2024-9-29 22:27
京公网安备11010202008513号 京ICP证101109号 京ICP备12012021号

ChaseDream 论坛

© 2003-2023 ChaseDream.com. All Rights Reserved.

返回顶部